[Rozgrzewka OM][MIX][Nierówności] Nierówności

Zadania z kółek matematycznych lub obozów przygotowujących do OM. Problemy z minionych olimpiad i konkursów matematycznych.
Regulamin forum
Wszystkie tematy znajdujące się w tym dziale powinny być tagowane tj. posiadać przedrostek postaci [Nierówności], [Planimetria], itp.. Temat może posiadać wiele różnych tagów. Nazwa tematu nie może składać się z samych tagów.
Awatar użytkownika
Premislav
Użytkownik
Użytkownik
Posty: 15687
Rejestracja: 17 sie 2012, o 13:12
Płeć: Mężczyzna
Lokalizacja: Warszawa
Podziękował: 195 razy
Pomógł: 5220 razy

Re: [Rozgrzewka OM][MIX][Nierówności] Nierówności

Post autor: Premislav »

Akurat QM-AM działa dla dowolnych liczb rzeczywistych, niekoniecznie dodatnich, tak że oddalam (i dowodzi się tego właśnie przez przejście z modułami i nierównością trójkąta). Wygląd lewej strony sugeruje w ogóle podejście geometryczne w tym kierunku:

Kod: Zaznacz cały

https://pl.wikipedia.org/wiki/Punkt_Fermata

ale jak powiedziała profesor Damek (a ja z upodobaniem cytuję), Ja nie jestem geometrą.


Nowe zadanie:
niech \(\displaystyle{ x_{1}, x_{2}\ldots x_{n} \ (n\ge 2)}\) będą liczbami rzeczywistymi większymi niż \(\displaystyle{ 1}\), dla których
\(\displaystyle{ |x_{i}-x_{i+1}|<1, \ i\in\left\{1,2\ldots n-1\right\}}\)
Proszę udowodnić nierówność
\(\displaystyle{ \frac{x_{1}}{x_{2}}+\frac{x_{2}}{x_{3}}+\ldots+\frac{x_{n}}{x_{1}}<2n-1}\)
Awatar użytkownika
Premislav
Użytkownik
Użytkownik
Posty: 15687
Rejestracja: 17 sie 2012, o 13:12
Płeć: Mężczyzna
Lokalizacja: Warszawa
Podziękował: 195 razy
Pomógł: 5220 razy

Re: [Rozgrzewka OM][MIX][Nierówności] Nierówności

Post autor: Premislav »

No cóż, nie wzbudziło zainteresowania (znowu to piszę, chyba kiepskie zadania daję). To jest zadanie 3. z Baltic Way 2010, rozwiązanie wzorcowe to po prostu indukcja po \(\displaystyle{ n}\). Liczyłem na to, że ktoś pokaże nieindukcyjne rozwiązanie, ale nic z tego.

Dla rozruchu nowe zadanie:
proszę udowodnić, że dla dowolnych liczb rzeczywistych dodatnich\(\displaystyle{ a_{1}, a_{2}, \ldots a_{n}}\) (przy czym przyjmujemy \(\displaystyle{ a_{n+1}=a_{n}, \ a_{n+2}=a_{2}}\)) zachodzi nierówność
\(\displaystyle{ \sum_{i=1}^{n}\frac{a_{i}^{2}}{a_{i+1}+a_{i+2}}\ge \frac{1}{2}\sum_{i=1}^{n}a_{i}}\)
H0t_Orange_B0i
Użytkownik
Użytkownik
Posty: 29
Rejestracja: 9 cze 2020, o 10:49
Płeć: Mężczyzna
wiek: 9
Podziękował: 9 razy
Pomógł: 4 razy

Re: [Rozgrzewka OM][MIX][Nierówności] Nierówności

Post autor: H0t_Orange_B0i »

To zadanie podobne jest do zadania z Asian Pacific Math Olympiad z roku 1991(które jest w niebieskiej książe Pana Pawłowskiego) , z tym że tam w mianowniku po lewej stronie było wyrażenie \(\displaystyle{ a_k + b_k }\) dla dwóch nieujemnych ciągów.
Łatwo można pocisnąc z nierówności Schwarza Caughiego w formie Engela.
Stwórzmy nowy ciąg.
\(\displaystyle{ y_i = a_{i+1} + a_{i+2} }\)
Wówczas lewa strona nierówności jest równa
\(\displaystyle{ \sum_{i=1}^n\frac{a_{i}^{2}}{y_{i}} }\)
Ponieważ oba te ciągi zawierają dodatnie elementy(gwarantuje nam to teza) to możemy skorzystać z nierównosći Schwarza Caughiego w formie Engela.
Wówczas otrzymujemy
\(\displaystyle{ \sum_{i=1}^n\frac{a_{i}^{2}}{y_{i}} \ge (\sum_{i+1}^n(a_i))^2 * 1/(\sum_{n=1}^n(y_i)}\)
Łatwo zauważyc że \(\displaystyle{ \sum_{i=1}^n(y_i) = 2\sum_{n=1}^n(a_i) }\) to oczywiście wynnika z definicji ciągu \(\displaystyle{ y_i}\)
podstawiając to od wcześniejszego równania i po reduckji otrzymujemy tezę.
Mam nadzieje że wszystko jest ok.
Awatar użytkownika
Premislav
Użytkownik
Użytkownik
Posty: 15687
Rejestracja: 17 sie 2012, o 13:12
Płeć: Mężczyzna
Lokalizacja: Warszawa
Podziękował: 195 razy
Pomógł: 5220 razy

Re: [Rozgrzewka OM][MIX][Nierówności] Nierówności

Post autor: Premislav »

Piszemy Cauchy'ego i dodatniość \(\displaystyle{ y_{i}}\) gwarantuje nam założenie zadania, a nie teza (na tezę nie można się powoływać przy jej dowodzie ofc), ale to detal, merytorycznie wszystko się zgadza. Możesz wrzucać następne zadanie.
H0t_Orange_B0i
Użytkownik
Użytkownik
Posty: 29
Rejestracja: 9 cze 2020, o 10:49
Płeć: Mężczyzna
wiek: 9
Podziękował: 9 razy
Pomógł: 4 razy

Re: [Rozgrzewka OM][MIX][Nierówności] Nierówności

Post autor: H0t_Orange_B0i »

Faktycznie drobne błędy redakcyjne były w moim poście.
Wrzucam kolejne.
Wyznacz najmniejszą wartość funkcji
\(\displaystyle{ f(x) = (x+a+b)(x+a-b)(x-a+b)(x-a-b)}\) jeśli \(\displaystyle{ a}\) i \(\displaystyle{ b}\) są dowolnym liczbami rzeczywistymi.
Awatar użytkownika
Premislav
Użytkownik
Użytkownik
Posty: 15687
Rejestracja: 17 sie 2012, o 13:12
Płeć: Mężczyzna
Lokalizacja: Warszawa
Podziękował: 195 razy
Pomógł: 5220 razy

Re: [Rozgrzewka OM][MIX][Nierówności] Nierówności

Post autor: Premislav »

Ukryta treść:    
H0t_Orange_B0i
Użytkownik
Użytkownik
Posty: 29
Rejestracja: 9 cze 2020, o 10:49
Płeć: Mężczyzna
wiek: 9
Podziękował: 9 razy
Pomógł: 4 razy

Re: [Rozgrzewka OM][MIX][Nierówności] Nierówności

Post autor: H0t_Orange_B0i »

Zrobiłeś szybciej niż myślałem choć trzeba przyznać ,że zadanie też nie było zbyt wymagające.
Oczywiście wszystko jest poprawnie ,możesz wrzucić nową nierówność.
Awatar użytkownika
Premislav
Użytkownik
Użytkownik
Posty: 15687
Rejestracja: 17 sie 2012, o 13:12
Płeć: Mężczyzna
Lokalizacja: Warszawa
Podziękował: 195 razy
Pomógł: 5220 razy

Re: [Rozgrzewka OM][MIX][Nierówności] Nierówności

Post autor: Premislav »

Może teraz coś takiego:
dla danych liczb dodatnich \(\displaystyle{ a,b}\) proszę wyznaczyć największą wartość, jaką przyjmuje wyrażenie
\(\displaystyle{ \frac{x_{1}\cdot x_{2}\cdot \ldots x_{n}}{(a+x_{1})(x_{1}+x_{2})(x_{2}+x_{3})\ldots(x_{n-1}+x_{n})(x_{n}+b)}}\)
przy czym \(\displaystyle{ x_{1}, \ x_{2}, \ldots x_{n}>0}\).
Awatar użytkownika
Premislav
Użytkownik
Użytkownik
Posty: 15687
Rejestracja: 17 sie 2012, o 13:12
Płeć: Mężczyzna
Lokalizacja: Warszawa
Podziękował: 195 razy
Pomógł: 5220 razy

Re: [Rozgrzewka OM][MIX][Nierówności] Nierówności

Post autor: Premislav »

Dobra, zmieniam zadanie, bo za długo wisi. Powyższe to zadanie z Wędrówek po krainie nierówności, robi się to z nierówności Huygensa (choć może istnieje inny sposób), po przekształceniu tego ułamka na przykład do takiej postaci (bardzo dużego wyboru tu nie ma, zasadniczo widzę dwie możliwości i w obu idea jest dokładnie ta sama):
\(\displaystyle{ \frac{1}{\left(\frac{a}{x_{1}}+1\right)\left(\frac{x_{1}}{x_{2}}+1 \right)\left(\frac{x_{2}}{x_{3}}+1\right)\ldots\left(\frac{x_{n-1}}{x_{n}}+1\right)\left(\frac{x_{n}}{b}+1\right)\cdot b}}\).
W zasadzie jeśli da się z Huygensa (tudzież z Höldera), to z samej AM-GM też się da, tylko że może to być dużo bardziej złożone.

Nowe zadanie:
proszę udowodnić, że gdy \(\displaystyle{ x,y,z\in\left(0, \frac{\pi}{2}\right], \ x+y+z=\pi}\), to \(\displaystyle{ \sin x+\sin y+\sin z>2}\).
TomciO
Użytkownik
Użytkownik
Posty: 289
Rejestracja: 16 paź 2004, o 23:38
Płeć: Mężczyzna
Lokalizacja: Kraków
Pomógł: 38 razy

Re: [Rozgrzewka OM][MIX][Nierówności] Nierówności

Post autor: TomciO »

Ukryta treść:    
Liczby rzeczywiste \(\displaystyle{ x, y, z }\) spełniają warunek \(\displaystyle{ x^2 + y^2 + z^2 = 2 }\). Dowieść, że \(\displaystyle{ x+y+z \leq 2 + xyz }\).
Awatar użytkownika
Premislav
Użytkownik
Użytkownik
Posty: 15687
Rejestracja: 17 sie 2012, o 13:12
Płeć: Mężczyzna
Lokalizacja: Warszawa
Podziękował: 195 razy
Pomógł: 5220 razy

Re: [Rozgrzewka OM][MIX][Nierówności] Nierówności

Post autor: Premislav »

Ukryta treść:    
Dodano po 2 dniach 36 minutach 40 sekundach:
mam też rozwiązanie bez żadnych przypadków:    
To wrzucę nowe zadanie, o czym poprzednio zapomniałem:
niech \(\displaystyle{ a>0, \ x_{1}, x_{2}\ldots x_{n}\in \RR^{+}, \ x_{1}+x_{2}+\ldots+x_{n}=1}\). Proszę wykazać, że
\(\displaystyle{ \sum_{k=1}^{n}\left(x_{k}+\frac{1}{x_{k}}\right)^{a}\ge \frac{\left(n^{2}+1\right)^{a}}{n^{a-1}}}\)
bosa_Nike
Użytkownik
Użytkownik
Posty: 1664
Rejestracja: 16 cze 2006, o 15:40
Płeć: Kobieta
Podziękował: 71 razy
Pomógł: 445 razy

Re: [Rozgrzewka OM][MIX][Nierówności] Nierówności

Post autor: bosa_Nike »

Poprzednie zadanie z sinusami:    
Bieżące:    
Dodano po 47 minutach 36 sekundach:
Kiks, równoważną postacią jest \(\displaystyle{ s>2R}\).
Awatar użytkownika
Premislav
Użytkownik
Użytkownik
Posty: 15687
Rejestracja: 17 sie 2012, o 13:12
Płeć: Mężczyzna
Lokalizacja: Warszawa
Podziękował: 195 razy
Pomógł: 5220 razy

Re: [Rozgrzewka OM][MIX][Nierówności] Nierówności

Post autor: Premislav »

Rozwiązanie jest jak najbardziej poprawne, zadajesz.
bosa_Nike
Użytkownik
Użytkownik
Posty: 1664
Rejestracja: 16 cze 2006, o 15:40
Płeć: Kobieta
Podziękował: 71 razy
Pomógł: 445 razy

Re: [Rozgrzewka OM][MIX][Nierówności] Nierówności

Post autor: bosa_Nike »

Najpierw konwencja. Rozważamy trójkąt \(\displaystyle{ ABC}\) (wierzchołek, kąt wewętrzny przy wierzchołku oraz miara tego kąta oznaczane są tym samym symbolem) o bokach \(\displaystyle{ a,b,c}\) (bok \(\displaystyle{ a}\) naprzeciw kąta \(\displaystyle{ A}\) itd.). Przez \(\displaystyle{ m_a}\) będziemy oznaczać długość środkowej boku \(\displaystyle{ a}\), przez \(\displaystyle{ w_a}\) będziemy oznaczać długość odcinka dwusiecznej kąta \(\displaystyle{ A}\) pomiędzy wierzchołkiem \(\displaystyle{ A}\) a punktem przecięcia z bokiem \(\displaystyle{ a}\). Analogicznie dla pozostałych boków/kątów.
Zadanie polega na udowodnieniu, że w dowolnym trójkącie nierozwartokątnym zachodzi nierówność podwójna $$\frac{(a+b)^2}{4ab}+\frac{(b+c)^2}{4bc}+\frac{(c+a)^2}{4ca}\le\frac{m_a}{w_a}+\frac{m_b}{w_b}+\frac{m_c}{w_c}\le\frac{a^2+b^2}{2ab}+\frac{b^2+c^2}{2bc}+\frac{c^2+a^2}{2ca}$$
Czy ta nierówność prawdziwa jest również dla dowolnych trójkątów rozwartokątnych?

Uwaga:
Nie jest wymagane wyprowadzanie wzorów na długość czewian \(\displaystyle{ m,w}\), jeśliby pojawiła się potrzeba użycia tych wzorów.
Awatar użytkownika
Premislav
Użytkownik
Użytkownik
Posty: 15687
Rejestracja: 17 sie 2012, o 13:12
Płeć: Mężczyzna
Lokalizacja: Warszawa
Podziękował: 195 razy
Pomógł: 5220 razy

Re: [Rozgrzewka OM][MIX][Nierówności] Nierówności

Post autor: Premislav »

Ukryta treść:    
ODPOWIEDZ